Week 2

अब Quizwiz के साथ अपने होमवर्क और परीक्षाओं को एस करें!

The nurse is caring for a client in labor who just received epidural analgesia. The nurse should monitor the client for which adverse effects? A. Hypertension B. Bladder distention C. Hypothermia D. Precipitous labor

Choice B is correct. Epidural analgesia may cause bladder distention. Bladder distention may cause pain that remains after initiation of the block and may interfere with fetal descent in labor. Choices A, C, and D are incorrect. Epidural analgesia is known to cause hypotension; thus, it is routine for a client to receive a preprocedural fluid bolus of isotonic lactated ringers. Hypothermia is unlikely with epidural placement. The fever associated with epidural analgesia is usually not caused by infection but may result from reduced hyperventilation and decreased heat dissipation. Epidural analgesia commonly slows the progression of the second stage of labor because it relaxes pelvic muscles.

The nurse is preparing to administer a prescribed medication to a client. The nurse should take which initial action? A. Verify the client's full name and date of birth B. Ask about any medication allergies C. Review the client's vital signs D. Review medications and potential interactions

Choice A is correct. When preparing to administer medications, the first nursing action is identifying the correct client. The other actions, such as verifying allergies, interactions, etc., cannot occur until the client's medical record is accessed, which starts with ascertaining the client's name and date of birth. Choices B, C, and D are incorrect. While verifying medication allergies, knowing what medications are ordered and their potential side effects is essential; the nurse's first action should be to verify that they have the right client via the two universal identifiers (name and date of birth). Vital signs are pertinent to the safe administration of certain medications (antihypertensive, etc.); however, the client's identity needs to be verified first.

What is the leading cause of cognitive impairment in old age? A. Stroke B. Malnutrition C. Alzheimer's disease D. Loss of cardiac reserve

Choice C is correct. Alzheimer's disease is the most common degenerative neurological illness and the most common cause of cognitive impairment. It is irreversible, progressing from deficits in memory and thinking skills to an inability to perform even the simplest of tasks. Choices A, B, and D are incorrect.

The nurse is caring for a client experiencing septic shock. The client's blood pressure is unstable and requires multiple intravenous (IV) vasopressor support. To determine if the client is meeting the treatment goals, the nurse anticipates the primary healthcare provider (PHCP) will order A. an arterial line. B. a chest tube. C. mechanical ventilation. D. an indwelling urinary catheter.

Choice A is correct. An arterial line will likely be ordered for a client receiving multiple intravenous (IV) vasopressors. The PHCP will place this line to obtain continuous blood pressure, which will be more accurate than intermittent blood pressure monitoring via an external cuff. The vasopressor dose is titrated to keep the blood pressure in the target range. An arterial line is also advantageous because arterial blood gas may be collected without requiring an arterial puncture. Choices B, C, and D are incorrect. There is no relationship between blood pressure instability and a chest tube or mechanical ventilation. Both devices would be effective for respiratory emergencies such as pneumothorax, severe hypoxia, or another respiratory ailment. An indwelling urinary catheter may be used in shock to determine if a client is responding to fluid replacement. However, this would be unhelpful in assessing the effectiveness of vasopressor support.

The nurse is assessing a 4-year-old client who was sent to the emergency department from urgent care. Assessment reveals tripod positioning, blue lips, mottled skin, inspiratory stridor, and excessive drooling. Vital signs are: Temp: 39 C HR: 188 RR: 46 O2: 82 % What is the priority action for the nurse to take at this time? A. Keep the child calm and call for emergency airway equipment B. Obtain IV access C. Assess the throat for a cherry red epiglottis D. Place the child on a high flow nasal cannula at 100% FiO2

Choice A is correct. Any child presenting with excessive drooling, distress, and stridor is highly suspicious of having epiglottitis. In addition, this client is already showing signs of circulatory compromise, including circumoral cyanosis and mottling. The priority nursing action in this emergency is keeping the child calm and calling for emergency airway equipment. The child is at risk of losing the airway, and the airway is always the priority. Choice B is incorrect. It is inappropriate to attempt to obtain IV access on a child suspected of epiglottitis before emergency airway equipment is available. The priority action at this time is keeping the child calm and calling for emergency airway equipment. Choice C is incorrect. It is inappropriate to assess the throat for a cherry red epiglottis. Although a cherry red epiglottis would confirm the diagnosis of epiglottitis, this child is at risk of losing airway patency. The priority action will be to protect the airway before IV access is attempted. Choice D is incorrect. Placing the child on a high-flow nasal cannula at 100% FiO2 is not the priority now. This answer probably sounded right because, you see, the SaO2 is 82%, and there is circumoral cyanosis. Oxygen sounds like the correct answer, but the airway is always prioritized. It will be challenging to deliver adequate oxygen without establishing airway patency. The child risks losing the airway, so all interventions must wait until emergency airway equipment is nearby. If anything upsets the child, the airway could spasm and obstruct completely, making intubation impossible. Therefore, keeping the child calm and calling for emergency airway equipment is the priority in epiglottitis patients.

When assessing self-perception, the nurse should ask the client which of the following? A. "How would you describe yourself?" B. "What gives you hope when times are troubled?" C. "Is your normal way of dealing with stress helpful to you?" D. "Are you having difficulty handling any family problems?"

Choice A is correct. Assessment of self-perception focuses on how the patient thinks of himself/herself. Choices B, C, and D are incorrect. Role addresses the daily duties or tasks. Values address important big concepts of life and death. Coping is in response to a stressor.

The nurse is caring for a client who is prescribed enoxaparin. Which laboratory value should the nurse monitor? A. Platelet count B. Activated Partial Thromboplastin Time (aPTT) C. International Normalized Ratio (INR) D. Troponin

Choice A is correct. Enoxaparin is a low molecular weight-based heparin (LMWH). One of the adverse events of enoxaparin is heparin-induced thrombocytopenia (HIT). This severe condition results in a 50% or more decrease in the platelet count while also causing thrombosis. Therefore, it is reasonable to monitor the platelet count after initiating enoxaparin. Choices B, C, and D are incorrect. Unlike unfractionated heparin, enoxaparin does not require aPTT monitoring (choice B). aPTT is appropriate to monitor a client receiving unfractionated heparin. An unfractionated heparin dose is titrated for an aPTT of 1.5 to 2.5 times the control (baseline) value to achieve the treatment effect. The INR (choice C) is a monitoring parameter for warfarin. Finally, troponin is a lab test for individuals with suspected myocardial infarction (choice D). The nurse should be concerned if the troponin was elevated, indicative of injury to the myocardium. None of these labs require monitoring during enoxaparin therapy.

A post-hemorrhoidectomy client is preparing for discharge to home. During the provision of discharge instructions, the nurse should highlight which of the following points? A. The proper technique for sitz bath B. Restricting fluid intake for 24 hours C. Laxative administration upon discharge D. Lying in a recumbent position

Choice A is correct. It is important that the client is instructed to perform a sitz bath appropriately, understanding that the operative site must be exposed to the warm, moist heat during each sitz bath. Specifically, the nurse should educate the client to perform sitz baths three to four times daily in the immediate post-operative period following a hemorrhoidectomy. Choice B is incorrect. There is no indication that this client should be placed on a fluid restriction for 24 hours, nor should such information be part of the client's discharge instructions. The National Academy of Medicine suggests an adequate intake of daily fluids of roughly 13 cups (104 ounces) for healthy men and 9 cups (72 ounces) for healthy women. Choice C is incorrect. Stool softeners are more likely to be prescribed for use upon discharge than laxatives, as laxatives are more likely to cause diarrhea and cause increased pain in the rectal region. Choice D is incorrect. The client would not be instructed to lie in a recumbent position upon discharge, as this position would pressure the surgical site. Conversely, the client would be encouraged to rest in a side-lying position, as this would reduce pressure on the surgical site and likely aid in preventing or minimizing discomfort.

The nurse is caring for a client receiving morphine sulfate for severe pain. The nurse should implement all of the following actions, except: A. Administer morphine only when the client complains of pain. B. Ensure naloxone is always available. C. Check the client's respirations before giving morphine D. Provide a high fiber diet.

Choice A is correct. Morphine should be given at times prescribed by the doctor to ensure adequate serum levels for optimum pain relief. Waiting to give it until the client experiences pain may lead to sub-optimal pain control. When specified by the physician, the nurse can provide morphine for breakthrough pain if the client complains of pain despite receiving round the clock morphine doses. Choice B is incorrect. Naloxone should always be available as an antidote to morphine. Choice C is incorrect. One of the significant side effects of morphine is respiratory depression. Checking the client's respirations before giving morphine should always be done. Choice D is incorrect. Morphine results in constipation for most clients. A high fiber diet should be given to prevent illness.

The nurse assesses a client with damage to cranial nerve III. Which finding would be expected? A. Ptosis B. Anosmia C. Uvula deviation D. Asymmetric facial movement

Choice A is correct. Ptosis, or eye drooping, occurs with cranial nerve III (oculomotor) lesions, myasthenia gravis, and Horner syndrome. Dysfunction of cranial nerve III is also associated with dilated pupil, absent light reflex, and impaired extraocular muscle movement. Choices B, C, and D are incorrect. Anosmia, or a decrease or loss in smell, occurs with dysfunction of cranial nerve I (olfactory), frontal lobe lesion, tobacco or cocaine use, allergic rhinitis, and upper respiratory infections. Uvula deviation to one side occurs with dysfunction of cranial nerve X (vagus), brainstem tumors, and neck injury. Unilateral facial movement occurs with dysfunction of cranial nerve VII (facial), central nervous system lesions such as stroke, and peripheral nervous system lesions such as Bell's palsy.

When performing a transcultural assessment, the nurse must: A. Determine which questions to ask the client based on the potential health effects of culture-based practices. B. Wait until the nurse-patient relationship is established before asking questions. C. Ask all questions for completeness of the assessment. D. Include all questions as part of an admitting assessment.

Choice A is correct. Since the list of suggested transcultural assessment questions is extensive, nurses can usually not conduct a complete assessment for each patient on admission to inpatient or outpatient care. Therefore, the nurse must determine which questions to ask based on the patient's symptoms, learning needs, and potential health effects of culture-based practices. A patient's behavior is influenced in part by his cultural background. Although certain attributes and attitudes are associated with particular cultural groups, not all people from the same cultural background share the same behaviors and views. When caring for a patient from a different culture, nurses need to be aware of and respect the patient's cultural preferences and beliefs. Failure to do so may cause the patient to feel that the nurse is insensitive and indifferent, possibly even incompetent. When performing a transcultural assessment, it is essential not to stereotype a patient based on what you believe their cultural beliefs/practices are. The best way to avoid stereotyping is to view each patient individually and find out their cultural preferences. Using a culture assessment tool or questionnaire can help the nurse discover these and document them for other healthcare team members. The American Nurses Association, the Joint Commission, the American Psychological Association, and other accrediting agencies direct nurses to acknowledge and address the cultural needs of patients. To facilitate this process, the U.S. Department of Health and Human Services and the Office of Minority Health of the U.S. Department of Health and Human Services published the National Standards for Culturally and Linguistically Appropriate Services in Health Care. Choice B is incorrect. The development of the nurse-patient relationship takes time. It is not appropriate to postpone assessment questions until the relationship is developed since this could cause neglect of immediate care needs. Choices C and D are incorrect. The transcultural assessment is extensive. Therefore, nurses usually cannot conduct a complete evaluation on admission.

A client with a peptic ulcer is prescribed sucralfate. Which statement by the client indicates an understanding of the medication? A. "I should take sucralfate at least 1 hour before meals and at bedtime." B. "I will avoid taking antacids completely while I'm on sucralfate." C. "I should take sucralfate right before meals and at bedtime." D. "I can expect immediate relief of my ulcer symptoms after taking sucralfate."

Choice A is correct. Sucralfate is a locally acting agent that combines with hydrochloric acid in the stomach in an acidic environment (pH less than 4) to make a cross-linking, thick compound that forms a barrier over the ulcer. Sucralfate is most effective when taken on an empty stomach to enhance its adherence to the ulcer site. It should be taken at least an hour before meals and at bedtime. Choice B is incorrect. This statement is not specific to sucralfate. If a client is also taking an antacid, it should be taken more than one half-hour before or after sucralfate oral liquid. Choice C is incorrect. Sucralfate should be taken on an empty stomach to enhance its adherence to the ulcer site. It should be taken at least an hour before meals and at bedtime. Choice D is incorrect. Sucralfate does not provide immediate relief; it acts by forming a protective coating over the ulcer. This helps the ulcer heal, and the onset of symptomatic relief may take several days or weeks.

The nurse is taking care of a pediatric client after an asthma attack. To promote comfort, the nurse instructs the client to assume which position? A. High Fowler's position B. Prone position C. Side-lying position D. Dorsal position

Choice A is correct. The High Fowler's position facilitates the breathing process for children after an asthma attack. Choices B, C, and D are incorrect. The prone, side-lying, and dorsal positions do not promote comfort in children after an asthma attack.

The first bodily area to be washed during a complete bed bath is the: A. Inner canthus of the eye B. Cheeks C. Forehead D. Chin

Choice A is correct. The first bodily area to be washed during a complete bed bath is the inner canthus of either of the client's eyes. Washing the client's eye from the inner to the outer canthus prevents secretions from entering the nasolacrimal duct. Additionally, when washing, each of the client's eyes should be washed using different sections of the bathing mitt or cloth for each eye to reduce the risk of infection transmission. After washing both of the client's eyes, the individual performing the complete bed bath should then ask whether the client prefers to use soap on their face. Based on the client's response, wash, rinse, and dry the client's forehead, cheeks, nose, neck, and ears. Choice B is incorrect. Although the cheeks are washed near the beginning of a complete bed bath, the client's cheeks are not the first body area to be cleaned. Due to infection control purposes, the client's eye (specifically, the inner canthus of the eye) is the initial bodily part cleansed during a complete bed bath. Choice C is incorrect. The forehead is not the first bodily area bathed during a complete bed bath. Due to infection control purposes, the client's eye (specifically, the inner canthus of the eye) is the initial bodily part cleansed during a complete bed bath. Choice D is incorrect. The chin is not the first area of the body cleansed during a complete bed bath. Due to infection control purposes, the client's eye (specifically, the inner canthus of the eye) is the initial bodily part cleansed during a complete bed bath.

A client is admitted to the behavioral health unit and diagnosed with acute mania. The nurse anticipates that the primary healthcare provider (PHCP) will prescribe which medication? A. Valproic acid B. Haloperidol C. Bupropion D. Fluoxetine

Choice A is correct. The gold standard for treating bipolar mania is mood stabilizers. Valproic acid (VPA) is a mood stabilizer and is efficacious in treating mania because it has a fast onset. Choices B, C, and D are incorrect. Haloperidol is a first-generation (typical) antipsychotic. The goal for acute mania (or bipolar depression) is mood stabilization. First-generation antipsychotics are not the mainstay of treatment for bipolar disorder. Antidepressants such as fluoxetine and bupropion would be detrimental for a client who is manic. This could exacerbate the mania.

What complication should the nurse monitor for during the immediate postoperative time following a thoracentesis? A. Pneumothorax B. Infection C. Dyspnea D. Aspiration

Choice A is correct. The most immediate postoperative risk factor is pneumothorax. Thoracentesis is a procedure in which a needle is inserted into the pleural space between the lungs and the chest wall. This procedure is done to remove pleural effusion (excess fluid) from the pleural space to help ease breathing. Some conditions, such as lung disease/infections, heart failure, and tumors may cause pleural effusion. All procedures have some risks. The risks of this procedure may include: air in the space between the lung covering (pleural space) that causes the lung to collapse (pneumothorax), bleeding, infection, and liver or spleen injury (rare). Symptoms of pneumothorax include dyspnea, chest pain, shortness of breath, and pain. Choice B is incorrect. Infection would not be evident during the immediate postoperative period. Choice C is incorrect. Dyspnea is a sign of pneumothorax. Choice D is incorrect. Aspiration is not a complication related to Thoracentesis.

The nurse has received the following prescriptions for newly admitted clients. The nurse should initially implement which of the following? A. initiate intravenous fluids to a client with anorexia nervosa. B. administer venlafaxine to a client with persistent depressive disorder. C. consult the social worker to begin discharge planning for a client. D. obtain a blood sample to evaluate a client's lithium level.

Choice A is correct. The priority is to attend to a client's physiological needs. Initiating intravenous fluids for a client with anorexia nervosa prioritizes over the other prescriptions because of the condition's ability to cause dehydration and severe fluid and electrolyte disturbances. Choices B, C, and D are incorrect. Administering venlafaxine for a client with a chronic depressive disorder is not the priority. This is a chronic problem, and acute problems come first. Consulting with the social worker for discharge planning is a low-priority task and is akin to providing discharge teaching. Obtaining a blood sample to evaluate a client's lithium level does not prioritize over a client-ordered intravenous fluid for a circulation problem.

The nurse supervises a novice nurse interviewing a client with a borderline personality disorder. Which client statement would demonstrate the client using transference? A. "You are just like my mother bothering me with these questions." B. "Instead of breaking objects, I have joined a kickboxing class." C. "I cannot be an alcoholic because I still go to work every day." D. "I told my boyfriend if he leaves me, I will kill myself."

Choice A is correct. This is an example of transference. In transference, the client's unconscious feelings toward a healthcare worker come to the surface that originally stems from someone else. For instance, if a client starts to have hostility towards the healthcare worker because they remind them of a family member with whom they had (or have) a negative relationship. The client bringing up their mother and pinning it on the healthcare worker exemplifies transference. Choices B, C, and D are incorrect. Instead of breaking objects, the client joining a kickboxing class demonstrates sublimination, a positive defense mechanism. The client denying their alcoholism is an example of denial, which is a common defense mechanism used in borderline personality disorder. The client stating she will kill herself if her boyfriend leaves her is an example of manipulation. This is commonly used in borderline personality disorder.

The nurse in the burn unit is preparing to perform a dressing change on a client with deep partial-thickness and full-thickness burns. The nurse understands that minimizing the client's pain during the dressing change is the top priority. All the following are appropriate nursing interventions, except: A. Administer a COX-2 inhibitor orally 30 minutes before the dressing change. B. Provide a clear explanation to the client about the procedure and how it will be performed. C. Changing the client's dressing carefully and handling burned areas gently. D. Let the client watch their favorite television show while dressing change is being performed.

Choice A is correct. This is an incorrect nursing intervention and, therefore, the correct answer to this question. For clients with deep partial-thickness and/or full-thickness burns, opioids (e.g., morphine sulfate) are the class of medication used to control pain. Opioid pain medication should be administered intravenously to the client at least 30 minutes before any dressing change. Choice B is incorrect. Providing a clear explanation to the client about the procedure and how it will be performed is an appropriate nursing intervention, as this helps to gain the client's cooperation and reduces their anxiety. Choice C is incorrect. Changing the client's dressing carefully and handling burned areas gently is an example of an appropriate nursing intervention performed to minimize pain experienced by the client during the dressing change. Choice D is incorrect. Letting the client watch their favorite television show while a dressing change is being performed is an example of an appropriate nursing intervention, as this is a method of providing the client with a distraction that can help decrease the client's pain.

The nurse is performing medication administration for four clients. Which client and medication should be administered first? See the image below. Client one: prednisone 10 mg PO daily for asthma exacerbation Client two: actaminophen 500 mg PO x 1 dose for fever Client three: magnesium oxide 250 mg PO daily for chronic alcoholism Client four: glarine insulin 15 units SubQ daily for diabetes mellitus A. Client one B. Client two C. Client three D. Client four

Choice A is correct. This medication is prescribed for a client with an asthma exacerbation which is an acute problem. Additionally, this acute problem deals with the client's breathing problem (asthma), prioritizing a fever, diabetes, and chronic alcoholism. Choices B, C, and D are incorrect. The client with a fever prescribed acetaminophen will require treatment but does not prioritize the acute respiratory ailment of an asthma exacerbation. Diabetes and alcoholism are chronic medical problems that would not require immediate administration of the prescribed medications when competing with a client with an acute asthma exacerbation.

The patient with history of right mastectomy is receiving maintenance IV fluids via peripherally inserted intravenous line in the left cephalic vein. The patient complains of pain at the IV site, and the nurse notes that the infusion has slowed and assesses swelling and erythema at the IV site. Which action should the nurse take first? A. Stop the infusion and remove the IV catheter B. Insert new IV in left intermediate basilic vein. C. Prepare the patient for PICC line placement. D. Elevate the right arm to reduce swelling.

Choice A is correct. This patient's IV site shows signs of phlebitis: redness, swelling, pain, and slowed infusion rate. The first priority action is to remove the current IV catheter to reduce the risk of further complications. Localized symptoms of phlebitis typically resolve after discontinuation of the catheter. Choice B is incorrect. Since this patient is not a candidate for IV access in the opposite arm due to a history of right mastectomy, the nurse should remove the current IV, and then attempt to insert a new IV proximal from the original site, but the current IV site should be discontinued first, prior to initiating any other interventions. Choice C is incorrect. This patient may be a candidate for PICC line placement if attempts to insert IVs at new sites are unsuccessful, but the current IV site should be discontinued first, prior to initiating any other interventions. Choice D is incorrect. Although phlebitis symptoms can be relieved by elevating the affected limb, applying a warm compress application, and administering analgesics, the current IV site should be discontinued first, prior to initiating any other interventions.

The nurse is teaching the parents of a client with cystic fibrosis. Which statement, if made by the parents, would require follow-up? A. "Chest physiotherapy should be done before giving bronchodilators." B. "The bronchodilator should be administered before strenuous activity." C. "My child may have trouble sleeping if the bronchodilator is given at night." D. "During a respiratory illness, my child should drink more water."

Choice A is correct. This statement requires follow-up. Bronchodilators should be administered before chest physiotherapy to enhance the mobilization of secretions, allowing them to be expelled. Choices B, C, and D are incorrect. These statements are correct and do not require follow-up. Bronchodilators should be administered before strenuous exercise to prevent respiratory distress. Specifically, short-acting bronchodilators such as albuterol need to be administered. Bronchodilators cause a discharge of catecholamines which may cause the client to have insomnia. Increasing the amount of non-caffeinated fluids is recommended during respiratory illnesses to assist with thinning secretions.

The nurse is caring for a client who has polycystic kidney disease (PKD). Which of the following would indicate the client is achieving treatment goals? A. Blood Pressure 128/63 mmHg B. Creatinine 2.3 mg/dL C. Proteinuria 2+ D. Sodium 132 mEq/L

Choice A is correct. Treatment goals for a patient with Polycystic Kidney Disease (PKD) include maintaining normotension, the glomerular filtration rate (GFR), and preventing sodium wasting, which is evidence of a decline in renal function. Hypertension is a cardinal finding in PKD, and if a client is achieving the treatment goals, they will maintain regulated blood pressure. Choices B, C, and D are incorrect. An elevated creatinine indicates that kidney function is declining. Proteinuria and hyponatremia are findings that would not suggest that the client is improving; rather, they would indicate that the client's renal function is declining.

A nurse cares for a client in the first trimester of pregnancy and notes that the client's serum potassium level is 2.9 mEq/L(3.5-5 mEq/L). Which of the following assessment findings is likely related to this lab finding? A. Alcohol consumption during pregnancy B. Hyperemesis gravidarum C. Lack of weight gain since the onset of pregnancy D. Food aversions

Choice B is correct. A serum potassium level of 2.9 mEq/L indicates hypokalemia. During pregnancy, hyperemesis gravidarum (vomiting) results in loss of fluids and electrolytes and is strongly associated with hypokalemia—additionally, loss of stomach acid results in metabolic alkalosis. Choice A is incorrect. Alcohol exposure in utero increases the risk of spontaneous abortion, decreases birth weight, and can cause fetal alcohol syndrome, a constellation of variable physical and cognitive abnormalities. Choice C is incorrect. Various factors may lead to a woman's lack of weight gain during the first trimester, but this lack of weight gain is unlikely to lead to such significant hypokalemia. Choice D is incorrect. Food aversions may cause the client to feel weak and nauseous during the first trimester but is unlikely to lead to hypokalemia.

The nurse preceptor is observing a newly hired nurse care for assigned clients. It would require follow-up by the nurse preceptor if the newly hired nurse is observed doing which of the following? A. Humidifies nasal cannula oxygen for a client with sarcoidosis. B. Secures a suprapubic catheter tubing to a client's inner thigh. C. Places a client with varicella-zoster in airborne and contact isolation. D. Suctions a tracheostomy for 10 seconds as they remove the catheter.

Choice B is correct. A suprapubic catheter should be looped and taped to the client's abdomen. Taping it to the abdomen decreases the tension on the tubing, decreasing its risk of dislodging. Choices A, C, and D are incorrect. Humidifying nasal cannula oxygen is appropriate, especially when used for the long term or at least 4 liters per minute. Varicella-zoster warrants airborne and contact precautions. Suctioning a tracheostomy for 10 seconds is appropriate as the catheter is removed in a twirling motion.

The nurse is preparing a client for a total laryngectomy. When developing a plan of care for this client, the nurse recommends a consultation from which healthcare provider (HCP)? A. Endocrinology B. Respiratory therapy C. Dermatology D. Infectious disease

Choice B is correct. A total laryngectomy is the removal of the larynx and surrounding lymph nodes. This is a significant procedure that requires the placement of a tracheostomy. The client will need an interdisciplinary approach to their care. A central figure for a client with a tracheostomy is a respiratory therapist collaborating with the nurse regarding tracheostomy management. Choices A, C, and D are incorrect. Healthcare professionals essential in managing client care following a total laryngectomy include respiratory and speech therapy. Psychiatry may be useful later in the rehabilitation process because the client may develop a disturbed body image. Endocrinology, dermatology, and infectious disease are not relevant to the care of a client who underwent a total laryngectomy.

A client is receiving allopurinol and asks what they should know about taking this medicine. The nurse would be most correct in stating which of the following? A. "Facial swelling is expected in the first few days of therapy." B. "Drink at least 3000 mL of water per day." C. "Do not eat while taking this medication." D. "This medication begins working immediately."

Choice B is correct. Allopurinol is prescribed to patients with gout or kidney stones and works by reducing the amount of uric acid produced by the body. Patients taking this medication should be encouraged to drink plenty of water, at least 3,000 mL per day. Choice A is incorrect. Facial swelling is not normal and may indicate an emergency reaction. Patients who experience swelling should seek medical attention as soon as possible. Choice C is incorrect. Eating with this medication is appropriate. Choice D is incorrect. This medication does not work immediately and may take a few months to reach full effectiveness.

The nurse is assessing a client with congestive heart failure. Which physical assessment finding should the nurse expect? A. Intermittent claudication B. S3 gallop C. Venous stasis ulcers D. Widened pulse pressure

Choice B is correct. An S3 gallop is an expected finding in heart failure. This is often an early manifestation of heart failure; it and this sound are best auscultated at the apex of the heart. Choices A, C, and D are incorrect. Intermittent claudication is a clinical feature of peripheral arterial disease. This causes a client pain as they ambulate and is relieved with rest. The pain may radiate to the ankle or buttock. Venous stasis ulcers are associated with long-term venous insufficiency; ulcer formed due to edema or minor injury to the limb; typically occurs over the malleolus. A widened pulse pressure (difference between the systolic and diastolic) is a feature of increased intracranial pressure, not heart failure.

The nurse is preparing to administer prescribed bumetanide to a client. Which clinical finding would indicate the desired outcome? A. Increase in central venous pressure B. Reduced cardiac preload and wall tension C. Decreased glomerular filtration rate D. Increase in systemic vascular resistance

Choice B is correct. The desired outcome for a loop diuretic is the following - Reduction of blood pressure - Reduction of pulmonary vascular resistance - Reduction of systemic vascular resistance - Reduction of central venous pressure - Reduction of left ventricular end-diastolic pressure Choices A, C, and D are incorrect. It is not desired for a diuretic to increase CVP. CVP is the measurement of right ventricular end-diastolic pressure. Diuretics reduce blood volume, thereby reducing this pressure. The goal of a diuretic is to reduce the CVP. It is not desired for a client to have their GFR reduced. The goal for all clients is to have a high GFR, as this is an indicator of renal health. Prolonged exposure to loop diuretics may reduce the GFR, especially at aggressive doses. An increase in systemic vascular resistance is also not the desired effect of diuretics. SVR is the amount of force exerted on circulating blood by the body's vasculature. Less blood volume by the diuretic = a decrease in SVR.

The clinic nurse notices bruising at multiple stages of healing on a two-year-old. The nurse also sees two burns on the toddler's trunk. What would be the most appropriate action for the nurse to take? A. Confront the child's parent(s)/caregiver(s) B. Call the local authorities or the designated state-specific child abuse hotline C. Recheck the child after two weeks D. Call the health care provider (HCP)

Choice B is correct. Bruises and/or burns present on a child are indicative of child abuse. Once the nurse suspects child abuse, the nurse is legally obligated to notify the local authorities or the designated state-specific child abuse hotline. Typically, following the alert made to local authorities or the designated state-specific child abuse hotline, referrals usually go to the state child welfare department and are assigned to a caseworker in an agency, such as Child Protective Services. After a referral has been made, a caseworker is assigned to investigate the report. Based on the investigation findings, the child is left in the home or temporarily removed. Choice A is incorrect. The nurse is not in a position to confront the two-year-old child's parent(s)/caregiver(s) regarding the bruises and burns the nurse has just observed, as this would compromise the nurse's safety. Choice C is incorrect. As noted above, once a nurse suspects child abuse, the nurse is legally obligated to notify the local authorities. Waiting two weeks and rechecking the two-year-old would violate the legal obligation of the nurse. Additionally, further harm may occur to the child due to this waiting period. Choice D is incorrect. Calling the health care provider (HCP) does not remove the two-year-old from danger. The nurse may need to contact the health care provider, but the nurse should call local authorities first.

The nurse observes unlicensed assistive personnel (UAP) give a bed bath using 4% chlorhexidine (CHG) wipes. Which observation requires follow-up? A. Uses one washcloth for washing each major body part B. Rinses the skin after bathing with the CHG solution C. Washes the client's face with warm water and mild soap D. Allows the CHG solution to dry on the client's skin

Choice B is correct. CHG is an effective antimicrobial agent that inhibits bacterial growth for 24 hours. The solution should not be rinsed off once it is applied, as it will leave a sticky residue. The sticky residue (sensation) is normal. Choices A, C, and D are incorrect. These observations do not require follow-up because they are appropriate. When using CHG in a bath basin of water, use one washcloth to wash each major body part. Then dispose of the cloth and use a new cloth for the next body part. CHG should not be applied to the face or the eyes. Only use warm water or mild soap, and water should be used on the face. CHG should be allowed to dry on the client's skin and should leave a residue-type sensation.

The nurse is performing a respiratory assessment of a patient with abnormal breathing patterns. The patient's breathing is rhythmic, yet has periods of apnea. The nurse would be most correct in charting this breathing style as: A. Neurogenic hyperventilation B. Cheyne-Stokes C. Apneustic D. Ataxic

Choice B is correct. Cheyne-Stokes is rhythmic breathing with periods of apnea usually caused by a metabolic issue or neurological problem. Choice A is incorrect. Neurogenic hyperventilation is regular and fast and indicates an issue in the pons or midbrain. Choice C is incorrect. Apneustic breathing is irregular and presents with apnea at the end and beginning of each breath. Choice D is incorrect. Ataxic breathing is utterly irregular in rate, depth, and rhythm.

The nurse is performing an initial assessment on a patient being admitted for acute pancreatitis. Which assessment data would support this diagnosis? A. Homan's sign B. Cullen's sign C. Hyperactive bowel sounds D. Kernig's sign

Choice B is correct. Cullen's sign refers to the bluish periumbilical discoloration/ecchymosis that is common in acute pancreatitis. The discoloration occurs due to blood-stained exudates seeping from the pancreas. Choice A is incorrect. A positive Homan's sign (pain in the calf with foot dorsiflexion) would indicate the presence of a DVT, not pancreatitis. Choice C is incorrect. A patient with acute pancreatitis would present with hypoactive (decreased) bowel sounds, not hyperactive. Choice D is incorrect. A positive Kernig's sign indicates possible subarachnoid hemorrhage or meningitis. It would not support the patient's acute pancreatitis diagnosis.

The nurse is caring for assigned clients. The nurse should immediately follow up with the client who A. has influenza and their most recent temperature was 102°F (39°C). B. is recovering from a thoracentesis and reports a nagging cough. C. reports reddish-brown sputum immediately following a bronchoscopy. D. has pulmonary tuberculosis and is wearing a surgical mask while ambulating to radiology.

Choice B is correct. Following a thoracentesis, the nurse must assess the client for the most common complication of pneumothorax. Manifestations of a pneumothorax that are concerning include a nagging persistent cough, increased heart and respiratory rate, dyspnea, and potentially a feeling of air hunger. The nurse must act quickly because the client's condition may deteriorate. Depending on the size of the pneumothorax, a chest tube may be needed. Choices A, C, and D are incorrect. A fever is common with influenza and would not necessitate the need for immediate follow-up. Following a bronchoscopy, reddish-brown sputum is expected because as the scope passes by the mucosa, it may irritate. Finally, no follow-up is necessary for a client with pulmonary tuberculosis wearing a surgical mask. This is an appropriate infection control measure. It is the healthcare worker that should wear the respirator (N95 mask).

The oncoming nurse is receiving a report on a pregnant patient with HELLP syndrome. This nurse knows that HELLP syndrome, a severe progression of preeclampsia stands for: A. Half Eclipsed Lipase Levels and Preeclampsia B. Hemolysis, elevated liver enzymes, and lowered platelets C. Hematocrit elevation, low lipase, and pancreatitis D. Hemoglobin, elevated lipids, and low plasma

Choice B is correct. HELLP syndrome stands for Hemolysis, elevated liver enzymes, and low platelets. HELLP syndrome is a condition in which hemolysis of the red blood cells occurs creating elevated liver enzymes and low platelets. Generally, complications are prevented by delivering the fetus as soon as symptoms develop. Choices A, C, and D are incorrect. These are not associated with HELLP syndrome.

A prenatal client is worried about her fetus' activity, so she performs a "kick count". She informs the nurse that while laying down, she felt ten kicks in one hour. The nurse should tell this client that: A. She'll need to come into the clinic and have a non-stress test performed. B. Ten kicks in an hour is a reassuring finding. C. She is dehydrated and should drink more water before re-trying the kick count. D. She should get up and walk for ten minutes and then re-try the test.

Choice B is correct. This is a reassuring finding. Ten kicks noticed during a 1 - 2 hour period are considered normal. Choices A, C, and D are incorrect. These interventions are not necessary.

The nurse is teaching a client about diabetes mellitus type I and exercise. Which statement, if made by the nurse, would be appropriate? A. Increasing exercise would increase insulin requirements B. Increasing exercise would decrease insulin requirements C. Insulin needs do not change with exercise D. Decreasing exercise would decrease insulin requirements

Choice B is correct. Increasing one's exercise would decrease insulin requirements in a client with type I diabetes, as exercise causes the client's blood glucose to decrease. While exercising, muscles require more glucose, and any circulating insulin present becomes more efficient in lowering glucose. Exercise has a variable effect on blood glucose, depending on the timing of exercise in relation to meals and the duration, intensity, and type of exercise. In clients with type 1 diabetes, exercise can lead to hypoglycemia. Therefore, the client's blood glucose should be monitored immediately before and after exercise. The target range for blood glucose prior to exercise should be between 90 mg/dL and 250 mg/dL (5 mmol/L to 14 mmol/L). Clients who experience hypoglycemic symptoms during exercise should be advised to test their blood glucose and ingest carbohydrates or reduce their insulin dose as needed to get their glucose slightly above normal just before exercise. Choice A is incorrect. A reduction in insulin doses is typically required to prevent exercise-mediated hypoglycemia in clients participating in moderately intense exercise for a prolonged duration (>30 minutes). Choice C is incorrect. In clients with type I diabetes mellitus, insulin needs will change based on the exercise patterns of the client. Choice D is incorrect. A decrease in the client's exercise routine does not correlate to a decreased insulin requirement in a client with type I diabetes mellitus. Type 1 diabetes is caused by an absence of insulin production due to autoimmune-mediated inflammation in pancreatic beta cells; therefore, the client will always require insulin. Following a decrease in the client's exercise routine, the client should be closely monitored to ensure the correct amount and type of insulin is being administered to obtain optimum blood glucose control.

The nurse has received a prescription for midazolam. Which of the following client findings requires follow-up with the physician prior to administering this medication? A. cocaine intoxication B. respiratory acidosis C. tonic-clonic seizures D. aggression

Choice B is correct. Midazolam is a benzodiazepine used for various indications, including aggression, seizure activity, and cocaine intoxication. Midazolam causes a reduction in respiratory rate because of its CNS depressant effect. If the client already has respiratory acidosis, administering a benzodiazepine, such as midazolam, would worsen the acidosis because the medication reduces the respiratory rate, further causing the retention of CO2. Choice A is incorrect. Cocaine intoxication manifests as psychotic behavior, psychomotor agitation, hypertension, tachycardia, and euphoria. Providing a CNS depressing medication, such as midazolam, is an indication of this medication. This is an appropriate indication for midazolam and does not require follow-up. Choice C is incorrect. During a tonic-clonic seizure, the client needs a benzodiazepine, such as midazolam, to terminate the CNS hyperexcitability. This is an appropriate indication for midazolam and does not require follow-up. Choice D is incorrect. Aggression and psychomotor agitation may benefit from a benzodiazepine, such as midazolam, to temper the client's behavior. This is an appropriate indication for midazolam and does not require follow-up.

A nurse is caring for a client receiving an intravenous oxytocin infusion for the induction of labor. The nurse notes the client's fundus has been contracting continuously for the past five minutes. An assessment of the fetal heart rate reveals 95 beats per minute. Which of the following should be the nurse's initial action? A. Place the client in a Trendelenburg position B. Stop the oxytocin infusion C. Administer oxygen via facemask D. Administer intravenous fluids at a high rate

Choice B is correct. Oxytocin is used to induce labor at or near full-term gestation and to enhance labor when uterine contractions are weak and ineffective. Here, when the nurse notices the client's fundus has been contracting continuously for the past five minutes, and an assessment of fetal heart rate reveals 95 beats per minute, the nurse should immediately recognize these symptoms as evidence of fetal distress. The priority action is for the nurse to stop the oxytocin infusion. Choice A is incorrect. The nurse should place the client in a side-lying position to aid in improving fetal circulation. Placing a client in a Trendelenburg position is used in situations of cord prolapse. Choice C is incorrect. Although the nurse should start oxygen to aid in fetal circulation, this is not the priority intervention. Choice D is incorrect. The infusion of intravenous fluid benefits clients in need of hydration; however, in this case, hydration is unnecessary as the cause of this client's problem stems from the infusion of oxytocin.

While working in the newborn nursery, you are called to L&D and asked to assign the APGAR score after birth. When you evaluate the infant at 1 minute of life, you find the following: cyanotic trunk and extremities, HR is 30 bpm, slight withdrawal when you pinch her foot, floppy muscles, and RR is 10 and irregular. What APGAR score do you assign? A. 1 B. 3 C. 5 D. 7

Choice B is correct. The APGAR score is 3. The infant gets 0 points for blue skin color all over, 1 point for an HR below 100 bpm, 1 point for a minimal response to stimulation, 0 points for absent muscle tone, and 1 point for a slow and irregular respiratory rate. This APGAR score indicates severe distress; therefore, the baby needs immediate action. Choices A, C, and D are incorrect.

The nurse is explaining immunizations to the parent of a pediatric patient. What type of acquired specific immunity would the Varicella vaccine fall under? A. Natural active immunity B. Artificial active immunity C. Passive natural immunity D. Passive artificial immunity

Choice B is correct. The Varicella vaccine contains a live chicken-pox virus. Artificial active immunity refers to the immunization of the specific antigen known to cause illness. This includes live and attenuated vaccines. Choice A is incorrect. Natural active immunity is achieved when there is contact with the infecting antigen through clinical infections. Choice C is incorrect. Passive natural immunity is when the benefits are passed down from the mother via transplacental and colostrum transfer. Choice D is incorrect. Passive artificial immunity refers to the short-term freedom that occurs upon the injection of serum antibodies from an individual who is immune to the body of someone who was not making these antibodies.

The nurse is talking to a group of female teenagers regarding the dangers associated with human papilloma virus. Which cancer mentioned by the group would indicate an understanding of the topic? A. Neuroblastoma B. Cervical cancer C. Osteoblastoma D. Osteosarcoma

Choice B is correct. The client with HPV has a higher risk for cervical and vaginal cancer. Choice A is incorrect. This type of cancer is not related to exposure to HPV. Choice C is incorrect. This type of cancer is not related to exposure to HPV. Choice D is incorrect. This type of cancer is not related to exposure to HPV.

A patient who is 2-days postoperative from right femoral popliteal bypass surgery complains of worsening right leg pain. Upon assessment, the RN notes swelling and ecchymosis at the incision sites. Which action would be the nurse's initial priority? A. Apply pressure to sites with sandbag B. Palpate pedal pulses C. Assess for signs of claudication D. Apply warm compress to incision sites

Choice B is correct. The most significant complications this patient is at risk for after the revascularization procedure are thrombus, hemorrhage, infection, and arrhythmias. Mild to moderate swelling, bruising, and pain at the surgical site are expected and typically resolve over time as the leaked blood is reabsorbed. The most important action would be to assess the patient's pedal pulses (distal to incisions). If pulses are intact, the nurse would then address the patient's complaint of worsening pain. Choice A is incorrect. These symptoms are expected following this type of surgery. Manual pressure would be appropriate if the patient was actively bleeding. Choice C is incorrect. Intermittent claudication is a cramp-like pain in the leg or buttock during activity due to poor blood supply. This is a sign of arterial disease, but not of postoperative complication, and would not be a priority for this patient. Choice D is incorrect. The RN should perform a focused assessment to rule out potential complications before implementing any interventions. Applying a warm compress may be helpful for reducing the patient's pain, but will also result in vasodilation which may increase swelling.

The psychiatric nurse is providing care for a patient who has just calmed down after exhibiting inappropriate behaviors related to bipolar disorder. The nurse knows that which of the following is the best way to help prevent another unseemly episode? A. Identify the consequences of the behavior. B. Assist the client in understanding triggering events or feelings that may have lead to the outburst. C. Ensure that the patient's safety is upheld. D. Offer the patient clear options to deal with their current behavior.

Choice B is correct. The psychiatric nurse would be most effective in preventing further inappropriate episodes by assisting the client in understanding what may have triggered the event. Choice A is incorrect. Identifying the consequences of inappropriate behavior would be a more appropriate intervention before the patient's response began escalating. Since this patient is calm, identifying values is not the most effective option to prevent reoccurring episodes. Choice C is incorrect. Ensuring the patient's safety is intact is always a priority but is a more appropriate action during the patient's episode of inappropriate behavior rather than while the patient is calm. Choice D is incorrect. A patient experiencing an episode of inappropriate behavior related to bipolar disorder is unlikely to absorb patient teaching. Teaching is best understood when the patient is calm and states readiness to learn.

The nurse is evaluating the lab test results of one of her prenatal clients. She is eight weeks along and has a hematocrit level of 36% (Male: 42-52% / Female: 37-47%) and hemoglobin of 11.7 gm/dL (Male: 14-18 g/dL / Female: 12-16 g/dL). These numbers are down from her pre-pregnancy H and H levels. The priority action of the nurse would be to: A. Call the client and request that she have her levels redrawn. B. Record these normal findings and confirm that the client is on a prenatal vitamin during her next visit. C. Report this abnormal finding to the doctor immediately. D. Notify the lab that these results are not normal and need to be re-assessed.

Choice B is correct. These results are typical and should be recorded as such. A drop from pre-pregnancy values is an expected phenomenon if they remain within or close to the normal range. Most women see a decrease in their hemoglobin and hematocrit levels during pregnancy. This phenomenon is known as physiological anemia and occurs as a result of increased plasma volume in the maternal bloodstream. It is essential to confirm that the client is taking prenatal vitamins. Demand for iron increases during pregnancy. Folic acid supplementation is necessary to prevent fetal neural tube defects. Prenatal vitamins will serve to address those needs. Normal hemoglobin in a pregnant client is > 11 g/dL. Normal hematocrit in a pregnant client is > 33%. Choices A, C, and D are incorrect. This mother will not need emergent care, and thus the doctor does not need to be notified regarding this expected drop from pre-pregnancy values. No lab values will need to be redrawn at this time.

The RN is caring for clients on a med-surg unit. Which result would warrant immediate intervention by the nurse? A. A blood glucose level of 250 in a type 2 diabetic being treated for pneumonia. B. A patient on a heparin drip with a 50% decrease in platelets over the past week. C. A type 2 diabetic client with A1C 10.5 complaining of tingling and numbness in the toes. D. An acute post-streptococcal glomerulonephritis client with a BP of 140/88 mmHg, proteinuria, and rust-colored urine.

Choice B is correct. This client is showing signs of heparin-induced thrombocytopenia (HIT): 50% decrease in platelets 5-10 days after heparin therapy was initiated. This is a thrombotic emergency and the nurse should assess the client, notify the physician, and discontinue the heparin drip. Choice A is incorrect. This client is being treated for pneumonia and is likely on antibiotics and corticosteroids. Both of these medications are known to increase blood glucose levels. This blood glucose result is high and the client may require a change in the insulin dose, but this would not be an emergency or the nurse's top priority. Choice C is incorrect. This client has an elevated A1C level (the ideal range is less than 7.0%). Hemoglobin A1C reflects blood sugar control over the past three months, so this would not be the highest priority. The patient complaining of tingling and numbness in the toes indicates peripheral neuropathy, a common problem in diabetic clients, mainly when blood sugars are poorly controlled. The nurse should determine what teaching/interventions the client needs to achieve better control of blood sugars and manage symptoms of neuropathy. Choice D is incorrect. This client is presenting with symptoms typical of acute post-streptococcal glomerulonephritis (APSGN): hypertension due to fluid retention, rust-colored hematuria due to upper urinary tract bleeding, and proteinuria due to decreased filtration. The symptoms that are expected are not the highest priority. Most clients with APSGN recover fully with conservative treatment and rest.

Which of the following would not be appropriate for the nurse to include in the teaching for a client with a diagnosis of acute low back pain? A. Smoking cessation B. Sleep in the prone position C. Use a firm mattress D. Bend at knees when lifting objects

Choice B is correct. This instruction is incorrect to recommend for this client, and therefore the correct answer to the question. Prone positioning results in excessive lumbar lordosis, which would increase the stress on the client's lower back. This client would benefit from sleeping in either a supine position or a side-lying position with a pillow between the knees and hips flexed. Choice A is incorrect. Nicotine has been shown to decrease circulation to vertebral disks. Smoking avoidance/cessation should be included in teaching for clients with low back pain. Choice C is incorrect. This client would benefit from the use of a firm mattress to provide adequate support to muscles, ligaments, and joints. Choice D is incorrect. The nurse should include information regarding proper body mechanics to limit stress on this client's lower back. The client should be instructed to avoid low back strain by bending at the knees (not the waist) when lifting objects and to stand up slowly with the item held close to the body.

The nurse is teaching a caregiver how to administer an injection of enoxaparin. Which statement, if made by the caregiver, would require further teaching? A. "I will give this injection in the abdomen." B. "I should give this injection at a 30 degree angle." C. "Acetaminophen is safe while taking this medication for any aches or pains." D. "A soft toothbrush should be used while taking this

Choice B is correct. This statement is incorrect and requires follow-up. Enoxaparin comes in prefilled syringes that are administered to the client subcutaneously. The appropriate angle for subcutaneous injection is 90 degrees or 45 degrees. 45 degrees would be an appropriate angle if the client did not have ample subcutaneous tissue. Choices A, C, and D are incorrect. These statements are correct and do not require follow-up. This injection should only be given in the abdomen via the subcutaneous route. Acetaminophen is the pain reliever recommended when compared to the alternative of aspirin or naproxen, which may potentiate the anticoagulant effects. Enoxaparin is an anticoagulant that does raise the client's risk of bleeding. A soft toothbrush is recommended.

While teaching a client who has recently begun a vegan diet, the nurse should highly recommend supplementing with which of the following vitamins? A. Vitamin C B. Vitamin B12 C. Vitamin A D. Vitamin D

Choice B is correct. Vitamin B12 is abundantly present in food products of animal origin. These include eggs, poultry, dairy products, fish, and meat. No strict vegetarian source has sufficient vitamin B12 to meet the recommended daily allowance (RDA). Vegans refrain from consuming all animal products, including eggs and dairy. Therefore, vegans are at a very high risk of developing vitamin B12 deficiency. Vegans should be counseled to consume alternative sources of vitamin B12 such as vitamin B12 supplements foods fortified with vitamin B12 ( fortified nutritional yeasts, fortified cereals) to reduce the risk of B12 deficiency significantly. Choices A, C, and D are incorrect. Vegans are generally not more prone to vitamin A and C deficiencies than non-vegans. Vegans consume plenty of fruits and vegetables. Vitamin A ( Choice C) is present abundantly in carrots, apricots, sweet potatoes, and dark green leafy vegetables ( spinach, kale, and collard greens). Vitamin C ( Choice A) is present abundantly in fruits ( orange, apple, kiwi, etc.) and vegetables ( bell peppers, brussel sprouts, broccoli, and so on). While vitamin D ( Choice D) is not abundant in a vegan diet, there are still some good vegan sources, including mushrooms, spinach, and bananas. Also, vitamin D can be abundantly obtained from sunlight. Vegans may be more prone to vitamin D deficiency than non-vegans. However, the vegans' highest risk is for vitamin b12 deficiency, and the nurse should prioritize this recommendation.

Following a persistent cough, chills, and fever, a client was admitted for a possible respiratory infection. The admission orders include a regular diet, vital signs every 4 hours, ampicillin 250 mg PO every 6 hours, and sputum culture. Before beginning antibiotic therapy, the nurse should perform which of the following? A. Provide the client a full meal B. Collect the sputum sample C. Assess the client's vital signs D. Assess the client's oxygen saturation

Choice B is correct. When caring for a client requiring a sputum culture, the sputum sample should be obtained before initiating antibiotic therapy. Obtaining the sputum sample prior to initiating antibiotic therapy allows for accurate detection of the organism(s) causing the infection through the sputum culture. Choice A is incorrect. Ampicillin should be given on an empty stomach (at least 30 minutes before or two hours after a meal). Choice C is incorrect. Overall, the assessment of a client's vital signs are an essential nursing action. Conversely, although obtaining new vital signs for any client prior to administering medication is always a sound practice, there is no specific indication or need to do so before administering oral ampicillin. Choice D is incorrect. Overall, assessing a client's oxygen saturation is an essential nursing action. Conversely, although obtaining a new oxygen saturation for any client before administering medication is always a sound practice, there is no specific indication or need to do so before administering oral ampicillin.

The nurse has just inserted an indwelling urinary catheter for a male client. The nurse plans on securing the catheter to the client's A. inner thigh. B. lower abdomen. C. outer thigh. D. medial thigh.

Choice B is correct. When securing an indwelling urinary catheter for a male, anchoring it to the lower abdomen (with the penis pointed upward) or upper thigh is appropriate. The catheter tubing should be secured to the lower abdomen or the upper thigh to prevent posterior urethral injury. Choices A, C, and D are incorrect. These anatomical locations are inappropriate for securing an indwelling catheter for a male. When securing an indwelling catheter for a female, it should be anchored to the inner thigh.

A nurse is assigned to care for a client with liver dysfunction and ascites and is ordered to measure the client's abdominal girth daily. To ensure accuracy, the nurse should utilize which landmark? A. Xiphoid process B. Umbilicus C. Iliac crest D. Symphysis pubis

Choice B is correct. With ascites, free fluid accumulates primarily in the abdominal cavity. As liver dysfunction worsens, ascites typically increase, increasing abdominal girth. When measuring abdominal girth, standard practice dictates using the umbilicus as the landmark to be utilized. The technique involves encircling the abdomen with a measuring tape at the level of the umbilicus. Choice A is incorrect. The xiphoid process, located at the inferior portion of the sternum, would not provide an accurate measurement of abdominal girth. Choice C is incorrect. The iliac crest, located at the superior border of the ilium and extending from the anterior superior iliac spine to the posterior superior iliac spine, would not accurately measure abdominal girth. Choice D is incorrect. The symphysis pubis, a joint located between the left and right pelvic bones, would not provide an accurate measurement of abdominal girth if used as a landmark.

The nurse is caring for a client diagnosed with attention deficit hyperactivity disorder (ADHD). The nurse should anticipate a prescription for which medication? A. Citalopram B. Risperidone C. Methylphenidate D. Carbamazepine

Choice C is correct. ADHD may be treated by psychostimulants such as amphetamines or methylphenidate. These medications work by projecting dopamine and norepinephrine in the front of the brain to ameliorate the symptoms of inattention, impulsivity, and hyperactivity. Choices A, B, and D are incorrect. Citalopram is a serotonergic drug used in the treatment of depressive and anxiety disorders. Risperidone is indicated for psychotic disorders such as schizophrenia. Carbamazepine is an anticonvulsant indicated for bipolar disorders as it has a mood-stabilizing effect.

Select the developmental age group that is accurately paired with the correct number of recommended hours of sleep per 24-hour period (including naps). A. Neonates: 14 to 16 hours of sleep per 24-hour period B. Infants: 12 to 14 hours of sleep per 24-hour period C. Toddlers: 11 to 14 hours of sleep per 24-hour period D. Preschoolers: 12 to 14 hours of sleep per 24-hour period

Choice C is correct. According to current recommendations, toddlers should have 11 to 14 hours of sleep (including naps) over an average 24-hour period. Choice A is incorrect. Neonates typically spend as much as 16 to 18 hours a day sleeping. Choice B is incorrect. During infancy, the recommendation is that infants should receive 12 to 16 hours of sleep during a 24-hour period (including naps). Choice D is incorrect. According to the current recommendations, preschool-age children should receive 10 to 13 hours of sleep per 24-hour period (including naps).

You are caring for a 17-year-old patient who has been taking isotretinoin (Accutane) for the past three months. The most critical assessment for this patient is: A. Improvement in the appearance of the skin B. Dry skin on the face C. Mood changes D. Problems remembering to take the medication

Choice C is correct. Accutane is a synthetic retinoid that is frequently prescribed for severe acne that does not respond to other topical and oral treatments. This medication is usually given for 4 to 6 months or until significant improvement is noticed. Effects can include dry skin and development in the appearance of the skin. However, there are also severe side effects that can develop. The FDA required that the labeling of Accutane be changed to add that there is a possible connection between Accutane and critical mood changes. Depression, irritability, altered sleep patterns, and suicidal ideation should be reviewed with the patient during every visit. Family members should be aware of the possibility of these problems. They should be instructed to watch for these symptoms and should call the physician immediately if issues are noted.

While assessing a newborn infant in the nursery, you observe bounding 3+ radial pulses and faint 1+ pedal pulses. You also notice that the feet are cold and pale, while the hands are warm and pink. Which cardiac defect do you suspect this infant has? A. Tetralogy of Fallot (TOF) B. Hypoplastic left heart syndrome C. Coarctation of the aorta (COA) D. Transposition of the great arteries

Choice C is correct. Coarctation of the aorta is a narrowing of the aorta near the ductus arteriosus. Because of this narrowing, there is increased blood flow to the upper extremities and decreased blood flow to the lower extremities. That causes the symptoms described in the question: bounding upper pulses, faint lower pulses, and overall better perfusion to the upper extremities. Choice A is incorrect. In the tetralogy of Fallot, four defects are combined - an overriding aorta, pulmonary stenosis, hypertrophy of the right ventricle, and a VSD. At birth, the nurse would appreciate a murmur and mild to severe cyanosis, depending on the case. The described symptoms do not fit the tetralogy of Fallot. Choice B is incorrect. In hypoplastic left heart syndrome, the left side of the heart is underdevelopment. The nurse would note cyanosis and murmur at birth, but the described symptoms do not fit hypoplastic left heart syndrome. Choice D is incorrect. In the transposition of the great arteries, the pulmonary artery leaves the left ventricle, and the aorta leaves the right ventricle. These infants are severely cyanotic at birth and need surgery early in life, but the described symptoms do not fit the transposition of the great arteries.

The nurse is caring for a client scheduled for electroconvulsive therapy (ECT). Which medication should the nurse question? A. Venlafaxine B. Esomeprazole C. Topiramate D. Lurasidone

Choice C is correct. ECT is a safe therapy that induces seizures theorized to release monoamines, which may assist in treating psychiatric illnesses such as major depressive disorder. If a client is taking the anticonvulsant topiramate, this will increase the seizure threshold and may attenuate the efficacy of ECT. Choices A, B, and D are incorrect. Antidepressant medications (such as venlafaxine) and antipsychotics (such as lurasidone) may be given concurrently with ECT. These medications may enhance the efficacy of the treatment. Proton pump inhibitors (such as esomeprazole) are typically given the day of treatment to prevent gastric reflux and aspiration.

While assessing a laboring mother during a contraction, the nurse notes a decrease in fetal heart rate from 150 to 120 bpm. The heart rate slows for about 10 seconds and increases back to 150 bpm as the contraction ends. Which of the following correctly classifies this observation? A. Late deceleration B. Moderate variability C. Early deceleration D. Marked variability

Choice C is correct. Early decelerations occur when the fetal heart rate decreases at the same time as a contraction. In this question, the nurse noted a decrease from 150 to 120 bpm with the contraction and then a return to baseline. This occurs due to the pressure of the head of the fetus on the pelvis or soft tissue, and no intervention is required by the nurse after an early deceleration. Choice A is incorrect. Late decelerations are a decrease in the fetal heart rate that occurs after a contraction. They are a non-reassuring sign on a fetal heart rate strip. In this question, the nurse noticed an early deceleration because it occurred with a contraction, not after. Choice B is incorrect. Variability on a fetal heart rate is defined as the fluctuations in the fetal heart rate from baseline. A moderate amount of variability is what is expected and is considered a reassuring sign. This question does not mention the variability of the fetal heart rate; instead, it notes an early deceleration. Choice D is incorrect. Variability on a fetal heart rate is defined as the fluctuations in the fetal heart rate from the baseline. Marked variability is a dramatically increased amount of these fluctuations. This question does not mention the variability of the fetal heart rate; instead, it notes an early deceleration.

The nurse in the emergency department (ED) is caring for a client experiencing agitation, anxiety, and hypertension. On assessment, the client has mydriasis and diaphoresis. The nurse suspects that this client may be intoxicated with which substance? A. Opioids B. Barbiturates C. Amphetamines D. Cannabis

Choice C is correct. Fever, mydriasis, agitation, paranoia, hypertension, and tachycardia are all manifestations of amphetamine intoxication. Substances producing this type of intoxication include amphetamines, methamphetamines, and cocaine, a central nervous stimulant. When a client experiences amphetamine withdrawal, they are likely to experience hypersomnia, fatigue, increased appetite, and dysphoria. Choices B, C, and D are incorrect. Opioid, cannabis, and barbiturate intoxication would cause a central nervous system depressant effect. The client would likely experience euphoria, slurred speech, psychomotor retardation, and delayed reaction(s). Examples of opioids include oxycodone and fentanyl. Examples of barbituates would consist of phenobarbital.

The nurse is caring for a client with appendicitis experiencing pain. Which pain relief method would be inappropriate for this client? A. Applying ice packs to the abdomen B. Practicing breathing exercises with the patient C. Using a heating pad D. Encouraging rest

Choice C is correct. Heat should not be applied to the abdomen of patients experiencing pain from appendicitis. Heat will cause vasodilation and increased blood flow to the appendix which may lead to rupture. A ruptured appendix puts the client at risk for a life-threatening condition known as peritonitis. Choices A, B, and D are incorrect. Applying ice packs to a patient's abdomen experiencing discomfort related to appendicitis is an appropriate non-pharmaceutical intervention. Using breathing techniques to work through appendicitis pain is an appropriate non-pharmaceutical intervention. Encouraging plenty of rest is an excellent way to prevent and manage pain from appendicitis.

The nurse is caring for a client presenting to the clinic with reports of abdominal cramping, bloating, and diarrhea after drinking milk. The nurse suspects that the client is at the highest risk for A. Pancreatitis B. Celiac disease C. Lactose intolerance D. Peptic ulcer disease

Choice C is correct. Lactose intolerance occurs because of insufficient amounts of the enzyme lactase or producing a less active form of the enzyme. Exposure to lactose causes a client to experience bloating, cramping, and diarrhea. Choices A, B, and D are incorrect. Pancreatitis may cause abdominal pain, vomiting, and dehydration. This condition is not linked to exposure to lactose, as either cholelithiasis or alcoholism causes pancreatitis. Celiac disease is when the client experiences gastrointestinal symptoms (nausea, vomiting, abdominal cramping) when exposed to gluten. Peptic ulcer disease is typically aggravated by spicy food that causes epigastric pain without diarrhea.

The nurse is taking care of a client that is status-post hand arthroplasty. The nurse should not include which nursing action to prevent complications? A. Encourage the client to exercise his fingers 10 times every hour, attempting full range of flexion and extension. B. Place the client's personal items within easy reach of the non-operative arm. C. Place the client's operative arm on a pillow to rest and keep it elevated. D. Encourage the client to use the non-operative arm as much as possible.

Choice C is correct. Placing the client's operative arm on a pillow produces pressure on the ulnar nerve. This should not be included in the client's care plan. The nurse should place the hand in a sling and suspend it from the bed. Choice A is incorrect. Finger exercises reduce edema and pain. This intervention should be included and must be implemented. Choice B is incorrect. Placing the client's items within reach of the non-operative arm fosters a sense of independence for the client and promotes safety. This should be included in the care plan. Choice D is incorrect. Encouraging the use of the non-operative arm fosters a sense of independence and self-care. This should be included in the care plan.

An emergency department nurse is caring for a 7-month-old infant with acute abdominal pain when the nurse begins to suspect intussusception. Which assessment finding would most support this suspicion? A. Black, tarry stool B. Ribbon-like stool C. Red, currant jelly-like stool D. Greasy, foul-smelling stool

Choice C is correct. Red, currant jelly-like stools are a characteristic of intussusception and are a result from a mixture of stool, blood, and mucus. Choice A is incorrect. Black, tarry stools are indicative of upper gastrointestinal bleeding. Choice B is incorrect. Ribbon-like stools may occur in some clients with Hirschsprung disease. Choice D is incorrect. Greasy, foul-smelling stools are a characteristic stool pattern for clients with cystic fibrosis.

What should the nurse do during assessment when a patient reports swelling in his ankles? A. Measure his ankles at their widest point. B. Ask the patient to elevate his feet to better visualize his ankles. C. Press fingers in the edematous area to evaluate for a remaining indentation after the nurse removes his/her fingers. D. Evaluate further for brown hyperpigmentation that is associated with venous insufficiency.

Choice C is correct. Reports of swelling require evaluation for pitting edema. This can be done by pressing fingers in the edematous area to evaluate for a remaining indentation after removing one's fingers. Choice A is incorrect. The nurse would not measure his ankles at their widest point. Choice B is incorrect. The patient's ankles should be evaluated for pitting edema. Elevating his feet is an intervention to prevent the pooling of fluid. However, it is not part of the assessment for edema. Choice D is incorrect. Hyperpigmentation is an indication of late-stage chronic venous insufficiency. Assessing for hyperpigmentation is not an immediate assessment necessary for the report of swelling of the ankles.

The nurse is teaching a client about newly prescribed tamsulosin. Which of the following statements should the nurse include? A. "This medication may turn your urine reddish/orange." B. "You will urinate more often with this medication." C. "Change positions slowly while you take this medication." D. "Avoid calcium-containing foods while on this medication."

Choice C is correct. Tamsulosin is an alpha-1 antagonist medication indicated in the treatment of benign prostatic hypertrophy. This medication causes vasodilation, and the biggest side effect is orthostatic hypotension. The nurse should educate the client to change positions slowly while taking this medication to reduce the risk of orthostasis. Choices A, B, and D are incorrect. Tamsulosin does not cause urine to change colors. This effect is more consistent with rifampin or phenazopyridine. The client will not urinate more often with this medication as it is not a diuretic. Rather, the client should urinate less often because of the ability for him to empty his bladder. Calcium-containing foods do not need to be avoided while a client takes this medication.

Which part of the laryngeal cartilage is a full circular ring and is the narrowest part of the airway in young children? A. Hyoid B. Arytenoid C. Cricoid D. Thyroid

Choice C is correct. The cricoid appears as a full circular ring and is the most narrow part of the airway. While intubating, it can be useful to place pressure on the cricoid to make the airway more comfortable to access. Choice A is incorrect. The hyoid is a semi-circle ring, not a circular ring. It helps support the tongue. Choice B is incorrect. The arytenoid muscle is at the back of the larynx and allows the vocal cords to work correctly. Choice D is incorrect. The thyroid is an organ that sits below the "Adam's apple" and is not a part of the airway.

The nurse is caring for a client in pain. The nurse asks the client which level of pain he is in, and the client says it's 1 out of 10. The nurse notices that the client grimaces every time he moves. What is the nurse's most appropriate action? A. Administer analgesics to the client. B. Move on to other patients. C. Ask the client about his grimacing with every movement. D. Encourage the client to watch his favorite TV show.

Choice C is correct. The nurse needs to assess the situation further when conflicting information is noted. Although the client states that his pain level is 1/10, his grimace at every movement tells otherwise. The nurse should validate her observations and make further assessments. Choice A is incorrect. The nurse needs to validate her observations first alongside what the client tells her to perform a more appropriate intervention. Choice B is incorrect. The nurse should not dismiss her observations about the client. Choice D is incorrect. Before performing interventions such as distraction, the nurse should validate her observations first.

Which of the following parenting styles is described as highly controlling, expecting to always be obeyed, and inflexible with the rules? A. Authentic B. Permissive C. Authoritarian D. Indifferent

Choice C is correct. The parenting style described is authoritarian. This parent is often described as the 'rigid disciplinarian'. They are highly controlling; they expect always to be obeyed and are still inflexible with the rules. Though these parents may have their child's best interests at heart, they do not support their growing autonomy. Instead, they expect to be obeyed without reason. Choice A is incorrect. Authentic not the parenting style described. An authentic parent sets reasonable limits on behavior, encourages the growing autonomy of their children, and promotes open communication about the rules. These parents would not be inflexible with the regulations; rather, they would explain why the rules work the way they do with their children to promote autonomy and open communication. Choice B is incorrect. Permissive is not the parenting style described. A permissive parent sets few or no restraints, gives their child complete unconditional love no matter the situation, as much freedom as they desire, places no limits, and offers very little guidance. While it is clear to everyone that these parents love their children very much, they are not constructive parents and do not support the growth of their children. Choice D is incorrect. Indifferent is not the parenting style described. An indifferent parent sets no limits for their child, lacks affection for their child, and is rather focused on their own life instead of that of the child. They are utterly indifferent to what their child does. This is very clearly ineffective for raising a healthy child.

The nurse is discussing information about advanced directives with a patient who expresses concerns, asking, "What if I change my mind about what I want?" What approach would you use to respond to the patient's care? A. Explain that the patient would have to file a new witnessed document in order to make any changes. B. Discuss the need to be very sure about his preferences, as the living will is a binding legal document. C. Assure the patient that he can change or revoke his advanced directives at any time. D. Advise the patient that changes could not be made during this hospital stay.

Choice C is correct. The patient may revoke either a living will or durable power of attorney at any time, and this can be done either verbally or in writing. Choice A is incorrect. The patient can revoke an advanced directive verbally; a newly written document is not required. Choices B and D are incorrect. While the 'living will' is a legal document, advanced directives can be easily changed at any time, just by saying so verbally, or with a newly written report.

Which of the following accurately summarizes the primary purpose of skin care and hygiene? A. Maintain skin sterility and prevent infection B. Prevent bodily odors by eliminating bacteria C. Protect the body's first line of defense D. Provide the client with comfort and well-being

Choice C is correct. The primary purpose of skin care and hygiene is to protect the body's first line of defense against infection, which is the skin. In addition to this primary purpose, skin care and hygiene also prevent bodily odors by eliminating skin surface bacteria, providing the client with comfort and well-being. Choice A is incorrect. Maintaining skin sterility and the prevention of infection is not a primary purpose of skin care and hygiene. The skin cannot be sterilized and can only be cleaned and disinfected using topical pharmaceutical agents. Choice B is incorrect. Although preventing bodily odors by eliminating bacteria is one purpose of skin care and hygiene, this is not the primary purpose. Choice D is incorrect. Although providing the client with comfort and well-being is one of the purposes of skin care and hygiene, this is not the primary purpose.

A newly hired nurse is caring for a client who is receiving prescribed total parenteral nutrition (TPN) therapy. The nurse preceptor should intervene if the newly hired nurse A. wears a surgical mask while changing the client's central vascular access dressing. B. obtains the client's capillary blood glucose every four to six hours. C. spikes and primes a new bag of TPN without an inline filter. D. continues the infusion via an infusion pump while the client is downstairs getting a computed tomography scan.

Choice C is correct. This action by the newly hired nurse requires follow-up because when a client receives TPN, it should have tubing with an in-line filter. The tubing should be changed every 24 hours when hanging a new bag of TPN. Choice A is incorrect. When performing a dressing change on the vascular access device, the nurse should instruct the client to wear a surgical mask while the nurse wears a surgical mask. This prevents droplet pathogens from contaminating the site. This action by the newly hired nurse does not require follow-up. Choice B is incorrect. Obtaining a client's capillary blood glucose every four to six hours is appropriate because the base solution of TPN is dextrose 40-50%. The complication of hyperglycemia is concerning because of its negative impact on the client's healing and risk for infection. This action by the newly hired nurse does not require follow-up. Choice D is incorrect. TPN should always be regulated via an infusion pump; if the client needs to leave the floor, it should be continued. If the TPN were abruptly discontinued, the client would be at risk for hypoglycemia because regular insulin is in the bag of the TPN. Regular insulin would peak without the supply of dextrose. If TPN were to be discontinued, it should be tapered.

The nursing student inserts an indwelling urinary catheter for a female patient prior to surgery. Which of the following would require immediate intervention by the RN? A. The patient states she feels the need to urinate. B. Patient reports a pinching sensation as the catheter is advanced. C. The student nurse notes resistance when inflating the balloon. D. The student separates the labia majora and labia minora with non-dominant hand.

Choice C is correct. This may indicate the balloon is within the urethra, not the bladder. If inflated within the urethra, the balloon may cause significant damage. Any complaints or nonverbal signs of discomfort or resistance is noted by the nurse during balloon inflation, are indications to stop this procedure immediately. Choice A is incorrect. The patient may feel the urge to void as the catheter is advanced through the internal urethral sphincter, this would not be a reason to stop the procedure. Choice B is incorrect. The student nurse should explain to the patient that she may feel pressure upon catheter insertion. A brief pinching sensation indicates the catheter is passing through the internal urethral sphincter and would not be a reason to stop the procedure. Choice D is incorrect. This action is appropriate. The student should use the non-dominant hand to position the patient and the dominant hand should remain sterile for insertion.

The nurse is working with an advocacy group to raise awareness about cystic fibrosis. Which statement best explains the condition? A. "It is an inherited disease that causes inflammation and hypersensitivity of the airway." B. "It is an infectious disease causing inflammation and fluid accumulation in the alveoli of the lungs." C. "It is an inherited disease causing excessive, thick mucus to build up in the body and cause blockages." D. "It is an acquired disease that causes inflammation and swelling of the epiglottis."

Choice C is correct. This statement correctly describes cystic fibrosis as an inherited disease causing excessive, thick mucus to build up in the body and cause blockages. Choice A is incorrect. Cystic fibrosis is NOT an inherited disease that causes inflammation and hypersensitivity of the airway. A disease that causes inflammation and hypersensitivity of the airway is asthma, not CF. Choice B is incorrect. Cystic fibrosis is NOT an infectious disease that causes inflammation and fluid accumulation in the alveoli of the lungs. Pneumonia is an infectious disease that causes inflammation and fluid accumulation in the alveoli of the lungs. Choice D is incorrect. Cystic fibrosis is NOT an acquired disease that causes inflammation and swelling of the epiglottis. Epiglottitis is a disease that causes inflammation and swelling of the epiglottis.

The nurse is preparing medications for the shift. Which of the following clients should the nurse prioritize for immediate medication administration? A. Digoxin to a client with atrial fibrillation B. Furosemide to a client with congestive heart failure C. Magnesium sulfate to a client with Torsades de pointes D. Labetalol to a client with a blood pressure of 160/100 mmHg

Choice C is correct. Torsades de pointes is an emergency because it is life-threatening and can progress to ventricular fibrillation and sudden cardiac death if not promptly treated. The nurse should immediately administer the prescribed magnesium sulfate to the client to prevent Torsades from degenerating into ventricular fibrillation. Choices B, A, and D are incorrect. All these medications are necessary for these clients and must be administered. However, the client with a more life-threatening condition (Torsades de pointes) should be prioritized.

The nurse is reviewing newly prescribed medications for assigned clients. Which of the following prescribed medications should the nurse question? A. Furosemide for a client with hyperparathyroidism B. Methimazole for a client with hyperthyroidism C. Hydrocortisone for a client with diabetes insipidus D. Prazosin for a client with pheochromocytoma

Choice C is correct. Treatment for diabetes insipidus includes medications such as desmopressin, thiazide diuretics, and anti-inflammatories. Hydrocortisone is a short-acting corticosteroid and is indicated in the treatment of adrenal insufficiency. This requires follow-up because DI is not treated with hydrocortisone. Choices A, B, and D are incorrect. Hyperparathyroidism causes hypercalcemia, and the treatment for hyperparathyroidism is a combination of 0.9% saline infusion followed by furosemide. Hyperthyroidism requires antithyroid medications such as methimazole or propylthiouracil. The classic manifestation of pheochromocytoma is hypertension, and treatment of this condition involves antihypertensive such as prazosin, an alpha-adrenergic blocker.

Total parenteral nutrition (TPN) is being considered for your client. Your client tells you, "My doctor is thinking about hyperalimentation, and I know nothing about it. Can you tell me what it is?". You should respond to this client's statement with: A. "Your doctor is thinking about total parenteral nutrition and not hyperalimentation." B. "Hyperalimentation is one kind of enteral nutrition that gives you feedings with a tube." C. "Hyperalimentation is one kind of parenteral nutrition that gives you feedings with a special IV line." D. "You should choose to have enteral nutrition and not accept hyperalimentation."

Choice C is correct. You should respond to this client's statement with, "Hyperalimentation is one kind of parenteral nutrition that gives you feedings with a special IV line." Parenteral nutrition, which is synonymous with hyperalimentation and IV hyperalimentation, provides the client with complete food when it is indicated for a client such as one who is adversely affected Choice A is incorrect. You should not respond to this client's statement with, "Your doctor is thinking about total parenteral nutrition and not hyperalimentation" because parenteral nutrition is hyperalimentation; parenteral nutrition is synonymous with hyperalimentation and IV hyperalimentation. Choice B is incorrect. You should not respond to this client's statement with, "Hyperalimentation is one kind of enteral nutrition that gives you feedings with a tube" because hyperalimentation is parenteral nutrition and not enteral nutrition. Choice D is incorrect. You should not respond to this client's statement with, "You should choose to have enteral nutrition and not accept hyperalimentation" because this is coercive and contrary to the client's right to make an informed decision about any or all care that is being considered.

Which of the following represents the appropriate daily caloric intake for an adolescent male who plays high school soccer 4 days a week? A. 1600 calories B. 2000 calories C. 2400 calories D. 2800 calories

Choice D is correct. 2800 calories (range 2800 to 3500 calories) are the appropriate caloric intake for an active adolescent male. Choice A is incorrect. 1600 calories will not be sufficient for an active adolescent male. This would be an appropriate caloric intake for a sedentary woman. Choice B is incorrect. 2000 calories would be an appropriate caloric intake for a sedentary older man and an active adult woman. Choice C is incorrect. 2400 calories would be an appropriate caloric intake for a sedentary adolescent, not for an active adolescent male.

During the initial assessment at a well-baby clinic, parents of a newborn ask the nurse when to begin introducing solid foods to their infant. The nurse replies that the recommended time solid foods should be introduced is: A. One year B. Two months C. Three months D. Six months

Choice D is correct. According to the American Academy of Pediatrics (AAP), an exclusive breastfeeding diet for a minimum of six months with the introduction of appropriate solid foods occurring around six months of age while breastfeeding continues. Beyond one year, breastfeeding should ideally continue for as long as both the infant and mother desire, although after one year, breastfeeding should complement a complete diet of solid foods and fluids. Therefore, the nurse should reply that the recommended time solid foods should be introduced to their child is six months of age. Choice A is incorrect. Infants should be introduced to solid foods well before one year of age. By twelve months, fine motor skills improve, allowing children to grasp pieces of food between two fingers. Choice B is incorrect. According to the American Academy of Pediatrics (AAP), breastfeeding should ideally occur exclusively for the initial six months. An infant's sucking reflex remains intact at two months, and the gastrointestinal and immune systems remain immature. Due to these factors, introducing solid food at two months of age would be inappropriate. Choice C is incorrect. According to the American Academy of Pediatrics (AAP), breastfeeding should ideally occur exclusively for the initial six months. Similar to the above answer, the infant's sucking reflex remains intact at three months of age, and the gastrointestinal and immune systems remain immature. Due to these aspects, introducing solid food at three months of age would be improper.

While rounding in the mental health unit, you are learning about specific phobias. You should be aware that ailurophobia is an unreasonable fear of: A. Social interactions B. Clowns C. Crowds D. Cats

Choice D is correct. Ailurophobia is best described as an unreasonable fear of cats. The psychiatric mental health treatment interventions for phobias are based on the specific type of phobia. For example, ailurophobia is usually treated with exposure therapy to the object or situation that is causing this unreasonable fear. Choice A is incorrect. A fear of social interactions is referred to as a social phobia. Social interaction phobias are typically treated with exposure therapy, antidepressants, or beta-blockers. Choice B is incorrect. The fear of clowns, which is referred to as coulrophobia, is typically treated with exposure therapy. Choice C is incorrect. The fear of crowds, which is referred to as enochlophobia, is also typically treated with exposure therapy.

Some intravenous therapies often consist of electrolyte replacement solutions. Select the electrolyte that is accurately paired with one of its functions. A. Phosphate: Operation of the sodium-potassium pump B. Potassium: Regulation of extracellular fluid (ECF) C. Chloride: Regulation of intracellular fluid (ICF) D. Calcium: Blood clotting

Choice D is correct. Among other functions, calcium plays a role in blood clotting. Other features of calcium include the formation of teeth and bones, nerve impulse transmission, and controlling muscular contractions. Choice A is incorrect. Phosphate does not control the operation of the sodium-potassium pump. Instead, phosphate plays a role in the production of DNA and ATP (adenosine triphosphate), and it also manages the acid-base balance of the body. Choice B is incorrect. Potassium regulates the intracellular fluid, not the extracellular fluid. Choice C is incorrect. Chloride regulates the extracellular fluid, not the intracellular fluid.

The nurse is caring for a client with Helicobacter pylori. The nurse should anticipate a prescription for which of the following medications? A. Dicyclomine B. Metoclopramide C. Valacyclovir D. Amoxicillin

Choice D is correct. Amoxicillin is an antibiotic that is commonly used to treat Helicobacter pylori infections. When treating this infection, this medication is often coupled with a proton pump inhibitor such as esomeprazole. Choices A, B, and C are incorrect. Dicyclomine is an antispasmodic medication used to treat gastrointestinal spasms, which are common in individuals with irritable bowel syndrome. Metoclopramide is a medication that causes gastric emptying and is used for nausea and vomiting. Valacyclovir is an antiviral indicated for herpes infections.

You are caring for a newborn born at term. On your assessment. You note that central cyanosis is present and persistent at five minutes after birth. You attach a pulse oximeter to the newborn. When determining whether or not the infant requires supplemental oxygen, you know that the expected oxygen saturation at 5 minutes after birth is: A. 65-70% B. 70-75% C. 75-80% D. 80-85%

Choice D is correct. At five minutes after birth, the expected SpO2 is in the 80-85% range. Regardless of the cyanosis, if the oxygen saturation is within this range, the infant probably does not need supplemental oxygen at this point. The American Heart Association and American Academy of Pediatrics suggest the following table for Target Pre-ductal Oxygen Saturation levels following birth. 1 min: 60-65% 2 min: 65-70% 3 min: 70-75% 4 min: 75-80% 5 min: 80-85% 10 min: 85-95%

The home health nurse is performing a follow-up visit on a child recently relocated to a new home following child abuse. The nurse anticipates that the child will likely demonstrate A. a willingness to explore new places. B. rapid social engagement with others. C. increased scholastic performance. D. reluctance or avoidance in social interactions.

Choice D is correct. Children subjected to abuse often have immediate psychosocial adverse effects such as acting aloof, being withdrawn, guarded, distrustful, and having difficulty engaging and maintaining social relationships. Choice A is incorrect. Children who have been abused are distrustful, resulting in their inability to explore new places. Children may prefer a predictable routine and not cope well with exploring new places. Choice B is incorrect. Social withdrawal and aloofness do not cause an individual who has been abused to develop rapid social engagement. These individuals appear guarded and may have minimal social interactions. Choice C is incorrect. An abused child will often appear fearful and withdrawn, which may result in a negative academic performance.

An 8-year-old pediatric client arrives complaining of swelling and pain in the bilateral knees. The child's parent informs the nurse, "The swelling came out of nowhere, and it just keeps getting worse." The initial impression is Lyme disease. Which additional question should be included during the assessment and history? A. "Have you noted any flank pain and a decrease in the urine volume?" B. "Has there been a fever of over 103 degrees over the last two to three weeks?" C. "Have you noticed any rashes on the palms or soles?" D. "Do you have headaches, malaise, or sore throat?"

Choice D is correct. Clients with Lyme disease often develop a musculoskeletal, flu-like syndrome consisting of malaise, fatigue, chills, fever, headache, stiff neck, myalgias, and arthralgias that may last for weeks. Symptoms are often nonspecific, resulting in the diagnosis being frequently missed if erythema migrans is absent. Choice A is incorrect. Urinary tract infections and/or a decrease in urine production are not symptoms commonly associated with Lyme disease. Choice B is incorrect. Any fever associated with Lyme disease is typically a low-grade fever. Choice C is incorrect. Erythema migrans, the hallmark and best clinical indicator of Lyme disease, is the first sign of the disease. It occurs in at least 75% of clients, beginning as a red macule or papule at the site of the tick bite, usually on the proximal portion of an extremity or the trunk (especially the thigh, buttock, or axilla), between 3 days and 32 days after a tick bite. The area expands, often with a clearing between the center and periphery resembling a bull's eye. Erythema migrans does not present as rashes on the client's palms and soles.

The nurse is planning a staff educational conference about indwelling urinary catheters. Which of the following information should the nurse include? A. Sterile gloves should be used to perform urinary catheter care. B. Urinary specimens may be collected from a catheter bag. C. You may irrigate a catheter with warm water for poor outflow. D. Daily use of soap and water should be used around the urinary meatus.

Choice D is correct. Daily cleaning of the urinary meatus with soap and water is recommended for catheter care. Sterile gloves do not need to be used for this process as it is a clean procedure. Soap and water is an acceptable practice for daily catheter care as alcohol, CHG, and other antiseptics may be highly irritating to the urinary meatus. Choices A, B, and C are incorrect. It is not necessary to utilize sterile gloves for catheter care as this is a waste of resources. Urinary specimens should not be collected from the catheter bag as this sample will be contaminated. If irrigating a urinary catheter is required, normal saline should be used as it is sterile. Water should not be used as it is not sterile and will cause cystitis.

The intensive care nurse (ICU) cares for a group of assigned clients. The nurse should initially follow-up with the client who is A. mechanically ventilated and not taking spontaneous breaths while in the assist-control (AC) mode. B. being treated for a flail chest, reporting chest pain with inhalation. C. noted to have gentle bubbling in the water seal chamber of their chest tube when coughing. D. receiving intravenous (IV) dopamine via a peripheral vascular access device and reports pain at the IV site.

Choice D is correct. Dopamine is a vasopressor and is indicated in the treatment of shock. Dopamine is a vesicant, and a significant adverse effect of dopamine is that it can extravasate and cause severe tissue damage. This emphasizes why this medication should be infused through a central line to prevent this adverse complication. The nurse should immediately attend to this client and stop the infusion. If extravasation is suspected, the nurse should stop the infusion and aspirate any remaining IV fluid from the catheter. Choice A is incorrect. The mechanically ventilated client has a protected airway, and the lack of spontaneous respirations explains why the client is receiving ventilation via assist control. Assist control provides full airway protection and delivers a preset number of breaths at a preset amount of tidal volume. The nurse does not need to follow up with the client as the ventilator provides appropriate treatment. Choice B is incorrect. A flail chest occurs from blunt force trauma and causes the client to have pain with inspiration. Another classic manifestation of a flail chest is paradoxical chest wall movement (inward thorax movement during inspiration, with outward movement during expiration). This is an expected finding. Choice C is incorrect. Gentle bubbling in the water seal chamber of their chest tube when coughing is normal and does not require follow-up. Continuous bubbling in the water seal chamber would require follow-up that suggests an air leak.

The nurse is caring for a client with heart failure. The patient is ordered a nitroglycerin patch to be attached. Which of the following nursing actions regarding the administration of a nitroglycerin patch is most relevant? A. Use a bare hand when putting the patch on the patient. B. Place the patch on the same spot every day. C. Place the client in the supine position with his feet elevated on a pillow. D. Instruct the client to rise slowly.

Choice D is correct. Patients under nitroglycerin therapy are at risk for postural hypotension. The client should rise slowly to avoid a sudden drop in blood pressure when standing up. Choice A is incorrect. The nurse should wear gloves when administering a nitroglycerin patch to avoid skin contact with the medication. Choice B is incorrect. The patch should be rotated to ensure optimum absorption through the skin. Choice C is incorrect. The purpose of nitroglycerin in heart failure is to dilate the venous circulation and trap the blood in the veins, decreasing the preload. Placing the client in the supine position and elevating his feet increases venous return, thus increasing preload. This defeats the purpose of nitroglycerin.

The nurse has attended a staff education program about managing clients with peripheral arterial disease. Which of the following statements by the nurse would require follow-up? A. "The client should engage in a daily exercise regimen." B. "Smoking cessation is an essential treatment goal for clients who smoke." C. "Resting in a recliner with the legs dependent should be recommended." D. "Devices that elevate the legs above the heart should be provided at discharge."

Choice D is correct. Peripheral arterial disease is caused by conditions such as hypertension, hyperlipidemia, and diabetes mellitus which cause atherosclerosis of the peripheral arteries. This impeded blood flow may cause the client to experience intermittent claudication (pain with ambulation that is relieved with resting). The client should be educated on self-management strategies, including sleeping or resting with the legs dependent (below the heart) to facilitate blood flow and not wearing constrictive clothing that may further impede blood flow. This statement requires follow-up because the client's legs should be below the heart to facilitate blood flow. Choices A, B, and C are incorrect. These statements indicate an effective understanding of managing PAD. Exercise is an effective part of the treatment plan as exercise promotes collateral circulation around vessels narrowed by plaque. Smoking cessation counseling is central in managing PAD as the vasoconstriction caused by nicotine is detrimental to the peripheral blood flow. Sleeping and resting with the legs dependent is recommended to facilitate distal blood flow.

The nurse is assessing a patient who reports intermittent tingling and numbness in bilateral lower extremities. Which intervention by the nurse would be most important to prevent injury for this patient? A. Perform Semmes-Weinstein monofilament test B. Refer the patient for a diabetic diet consult C. Obtain an order for Gabapentin D. Teach the patient about appropriate footwear

Choice D is correct. Peripheral neuropathy puts the patient at increased risk for traumatic injury and tissue breakdown since the patient may not notice early skin damage due to altered sensation. Of the options provided, educating the patient on proper footwear is the only action that aims to prevent injury related to the patient's altered sensation in the feet. Choice A is incorrect. The Semmes-Weinstein monofilament test is an appropriate way to test for sensation in the feet and is used to identify risk for neuropathic ulceration, but this assessment tool is not a preventative action. Choice B is incorrect. Diabetes is a common cause of peripheral neuropathy, but the question does not provide any information that indicates this patient is diabetic, and would not specifically address promoting the patient's safety. Choice C is incorrect. Gabapentin is used to improve neuropathic pain and may be appropriate for this patient, but would not directly prevent injury.

The patient who is two days postoperative cesarean section complains of right shoulder discomfort. Which action should the nurse take first? A. Administer PRN analgesic. B. Obtain STAT EKG. C. Encourage ambulation. D. Discuss the pain with the patient.

Choice D is correct. Shoulder pain may occur following a cesarean section due to gas or referred pain from the surgery. The nurse should assess the patient's pain to determine the cause before administering medications or other interventions. Choice A is incorrect. The nurse should first assess the patient's pain to determine the cause before administering pain medication. Choice B is incorrect. The nurse should first assess the patient's pain. If assessment data indicates the patient's pain is cardiac, an EKG may be indicated. Choice C is incorrect. Ambulation may help if the patient's pain is related to gas/indigestion, but the nurse should first assess the patient's pain before implementing this intervention.

The nurse is reviewing newly prescribed medications for assigned clients. Which of the following prescribed medications should the nurse question? A. captopril for a client with congestive heart failure B. metoprolol for a client with multiple premature ventricular contractions (PVCs) C. verapamil for a client with atrial fibrillation D. spironolactone for a client with end-stage renal disease

Choice D is correct. Spironolactone is a potassium-sparing diuretic and is primarily indicated in treating essential hypertension. The potential (significant) issue is that clients with end-stage renal disease commonly have hyperkalemia because of the significantly reduced glomerular filtration rate and rely on dialysis to remove nitrogenous waste, water, and electrolytes. It would be detrimental for a client with ESRD to receive spironolactone because this medication will raise serum potassium levels that are already high. This prescription requires follow-up. Choices A, B, and C are incorrect. Captopril is an ACE inhibitor. ACE inhibitors are highly recommended in managing heart failure because they decrease the complication of cardiomegaly, which may cause a further reduction in cardiac output. Metoprolol for a client with multiple premature ventricular contractions (PVCs) is appropriate. PVCs are generally benign and cause the ventricles to empty before they are filled. This may give the client a palpitation or flutter sensation. The nurse should identify the underlying cause for multiple PVCs, such as hypokalemia, hypomagnesemia, or central nervous stimulants. If the cause cannot be identified, beta-blockers may slow the heart rate to prevent premature firing. Atrial fibrillation is an irregularly irregular arrhythmia with two treatment goals: 1. prevention of stroke 2. rate control between 60-100 beats per minute. Verapamil is a calcium channel blocker that may be used for an individual with atrial fibrillation; this will help maintain rate control. Another medication that may be used instead of verapamil would be diltiazem or amiodarone.

Select the fact about non-steroidal anti-inflammatory drugs (NSAIDs) that is accurate. Which of the following is true about NSAIDs? A. Vary significantly and greatly in terms of their analgesic effects among the different medications in this classification of medications. B. Vary little in terms of their anti-inflammatory effects among the different medications in this classification of medications. C. Cannot be given with an antacid medication because it will interact with the NSAID in terms of its effectiveness. D. Mortality was lesser among patients receiving NSAIDs compared to opioids.

Choice D is correct. The World Health Organization ( WHO) recommends using NSAIDS and acetaminophen to treat pain before considering opioids. Long term use of NSAIDS can lead to life-threatening complications, such as gastrointestinal system bleeding and renal dysfunction, with long-term use. However, available evidence indicates that the all-cause mortality was higher in patients receiving opioids than other pain medications. Choice A is incorrect. There is little difference between the various NSAID medications in terms of their analgesic effects. Choice B is incorrect. There are significant differences among the various NSAID medications in terms of their anti-inflammatory effects. Choice C is incorrect. It is recommended that an antacid medication is given when the client is taking non-steroidal anti-inflammatory drugs (NSAIDs) to prevent gastrointestinal bleeding.

The nurse is developing group therapy sessions on substance use disorders. The nurse develops weekly topics and plans to host the sessions in a community center. The nurse is in which phase of therapeutic group development? A. Working B. Orientation C. Termination D. Planning

Choice D is correct. The nurse is in the planning phase, and the crux of this phase is to identify the purpose of the group, its objectives, individuals who may attend, and weekly topics. Choices A, B, and C are incorrect. The working phase is when the group performs the work, such as communicating with others, doing therapeutic exercises, and strengthening their rapport. The orientation phase is when the nurse ensures that group members feel welcome and understand the group's purpose. Termination is when the nurse recaps the group's successes and future goals.

The nurse is caring for a client who sustains a traumatic amputation of two fingers. The nurse should A. apply direct pressure to the severed fingers and wrap them in gauze. B. irrigate the amputated fingers with sterile saline. C. place the amputated fingers directly on ice. D. wrap the fingers in gauze, put it in a plastic bag, and then place the bag in ice water.

Choice D is correct. The nurse should wrap the amputated digits in a dry, sterile gauze or clean cloth. Put the wrapped part in a plastic bag or waterproof container. Place the plastic bag or waterproof container on ice. The nurse should not place the digit(s) directly on ice. The digit(s) should be insulated with gauze and a plastic bag to avoid nerve injury. Choice A is incorrect. Direct pressure should be applied to the amputated area (the hand), not the fingers. Choice B is incorrect. Irrigating the amputated fingers with sterile saline is inappropriate. The nurse needs to focus on preserving the digit(s) so they can be reattached. Choice C is incorrect. The amputated fingers should be placed in a waterproof plastic bag and placed in ice water, not directly on ice. Ice should not come directly in contact with the amputated part because it can cause ice burns, and the fingers may not be able to be sewn back.

The nurse is reviewing laboratory data for a client taking prescribed phenytoin. The client's phenytoin level is 12 mcg/mL(10-20 mcg/mL). Which action should the nurse take next? A. Evaluate the client for non-adherence. B. Instruct the client to skip the next scheduled dose. C. Assess the client for phenytoin toxicity. D. Document the result as within normal limits.

Choice D is correct. The therapeutic phenytoin level is 10-20 mcg/mL. A phenytoin level of 12 mcg/mL is considered therapeutic. Choices A, B, and C are incorrect. The phenytoin level is normal and does not require the client to be evaluated for non-adherence. Further, skipping the next dose and assessing the client for toxicity would be inappropriate because the level is therapeutic.

The occupational health nurse was called to see a client who sustained injuries from a light bulb explosion. On assessment, the nurse notes that a piece of glass was lodged in the client's eye. The initial nursing intervention should be which of the following? A. Attempt to carefully remove the glass from the eye B. Reassure the client that everything is okay C. Administer a sedative for pain relief D. Advise the client to remain in a sitting position until a specialist arrives

Choice D is correct. To prevent the intraocular pressure (IOP) from increasing, the client should be advised to remain seated, as the lying position may increase IOP and cause the glass to advance further into the eye. The nurse should also recommend the client rest and avoid unnecessary movement until a specialist (typically an ophthalmologist on call) arrives to evaluate the client. Choice A is incorrect. In cases of penetrating wounds, a nurse should never attempt to remove the foreign body (such as this piece of glass from a lightbulb) from the client's eye, as the object could tear or rupture the internal ocular structure, resulting in further damage, permanent damage, and/or blindness. Choice B is incorrect. The nurse may reassure the client that medical help is on the way and remain with the client until the specialist arrives. Conversely, the nurse should avoid providing false reassurance with statements broadly informing the client, "[e]verything will be okay," as these statements undermine the nurse's credibility and hamper any potential attempt by the client to express concern. False reassurance is inappropriate. Choice C is incorrect. The nurse must not administer any sedative unless ordered by a health care provider. Additionally, sedatives are not the class of medications typically utilized for pain relief.


संबंधित स्टडी सेट्स

Combo with "Strategic Management Chapter 5 - T/F" and 1 other

View Set

Sociology 213 Test 5 Ch. 10 Marriage and Communication in Intimate Relationships Ch. 12 Raising Children: Promises and Pitfalls

View Set

Chapter 6 - Trigonometric Formulas and Equations

View Set

Map Reading & Land Navigation (2)

View Set

M45.1: Budget Deficits and the National Debt

View Set